Your boss sends you to the hardware store to buy tie down rope. He tells you to buy the best deal. What is the savings per unit between the best deal and worst deal?

Answers

Answer 1

Answer:

Explain more

Step-by-step explanation:

You need to explain more like add an image or tell us how much much money is a rope.


Related Questions

**The domain of f(x) = 3x + 2 is
{-1 in the range?

Answers

Answer:

Yes

Step-by-step explanation:

Graph this function on your calculator and you will see an upwards graph, if the range (y), -1 can be mapped on the graph, it is within the range.

what is the psychology of a middle school student compared to a elementary compared to a high school compared to a college how does the brain change

Answers

Our brain's capacity increases day by day with respect to our growth of our bodyAs older the student be the sharper theirs brain becomes.A middle school student has a less equipped brain capable to handle their studies.But a hugh school student has more sharper brain than the middle school student.

The middle school is purchasing shoes for its basketball team. The cost of
each pair of shoes is $42. The total cost, t,is proportional to s,the number
of pairs of shoes ordered. Which of the following equations models this
proportional relationship? *

Answers

Answer:

9s = 378t      

is the same as  (9 x 42)   etc...

Step-by-step explanation:

I don't have the choice of answers of what is being said as proportional but an example is here below of what numbers are multiples of 42 and therefore found its proportion this way.

10s = 420t       (10 x 42)

9s = 378t         (9 x 42)   etc...

8s = 336t

7s = 294t

6s = 252t

5s = 210t    (5 x 42)

4s = 168t    etc....

¿Cuántas hipotenusa tiene el triangulo rectángulo?

Answers

Answer:

Step-by-step explanation:

Only one hypotenuse and it lies opposite to the right angle.

what is 7 1/2 divided by 5 5/8

Answers

Answer:

1.33

Step-by-step explanation:

9514 1404 393

Answer:

  4/3

Step-by-step explanation:

If you're familiar with decimal equivalents (or even if you're not), you can use your calculator to compute the quotient:

  7.5/5.625 = 1.333333333...

The rational number equivalent of the result is 4/3.

__

If you like to work directly with the rational numbers, you can write the ratio as ...

  (7 1/2)/(5 5/8) = (7 4/8)/(5 5/8) = (60/8)/(45/8) = 60/45 = 4/3

12 1/2, 3 3/4 what is the distance between the two

Answers

Answer:

Estimate the following (by rounding off to nearest hundred (a)6,941 (b)6,320 (c) 416 (d)126 (e)109 (f)202 (g) 146 (h)7,699 (I)8,311 (j)986

Step-by-step explanation:

Answer: 8 3/4

Explanation:
12 1/2 can be written as 12 2/4. One way of solving this problem is to convert each value into an improper fraction and then subtract.

12 2/4 = 50/4
3 3/4 = 15/4

50/4 - 15/4 = 35/4
35/4 can be simplified into 8 3/4

Write the scale without units: 1 cm:1 km

Answers

Step-by-step explanation:

scale = 1/100000 .

hope this helps you.

A large tank is partially filled with a solution. The tank has a faucet that allows solution to enter the tank at a rate of 16 3/4 liters per minute. The tank also has a drain that allows solution to leave the tank at a rate of 19 4/5 liters per minute. (a) What expression represents the change in volume of solution in the tank in 1 minute? (b) Simplify and give your answer as a simplified mixed number. (c) What is the change in volume of the solution after 10 minutes? Show necessary work. Give your answer as a simplified mixed number.

Answers

The change in volume per minute and the total change in the volume of the solution after 10 minutes will be [tex] - 3 \frac {1}{20} \: litres [/tex] and [tex]- 30 \frac {1}{2} \: litres [/tex] respectively.

Entry rate = [tex] 16 \frac{3}{4} [/tex] per minute

Discharge rate = [tex] 19 \frac{4}{5} [/tex] per minute

Change in the volume of solution in the tank per minute can be calculated thus :

Change in volume = (Entry rate - Discharge rate) × time

[tex] 16 \frac{3}{4} - 19 \frac{4}{5} = \frac{67}{4} - \frac{99}{5} = \frac{(335 - 396)}{20} = \frac{-61}{20} = - 3\frac{1}{20} \: litres \: per \: minute [/tex]

The total change in the volume of solution in the tank after 10 minutes can be calculated thus :

Change in volume per minute × time

[tex] - \frac{61}{20} \times 10 = - \frac{610}{20} = - 30 \frac {1}{2} \: litres [/tex]

Therefore, the solution in the tank decreases by [tex] 30 \frac {1}{2} \: litres [/tex] after 10 minutes

Learn more :https://brainly.com/question/18796573

Jordan has completed 5/8 of his homework assignment. What is the decimal equivalent of the amount of the assignment he has completed?

Answers

5.8 I think if I’m wrong sorry

Answer:Try to see what you have to do and see if you have to divide and add subtract and multiply

Step-by-step explanation:

five times x equals one

Answers

Answer:

x=0.2

Step-by-step explanation:

1 / 5 = 0.2

therefore

5 x 0.2 = 1

26-14-(-6)+(-7)-11

Please help

Answers

[tex] = 26 - 14 - ( - 6) + ( - 7) - 11 \\ = 26 - 14 + 6 - 7 - 11 \\ = 26 + 6 - 14 - 7 - 11 \\ = 32 - 14 - 7 - 11 \\ = 0[/tex]

Answer:

0

Step-by-step explanation:

G

E

M

D

A

S

26 - 14 - (-6) + (-7) - 11

26 - 14 = 12

12 - (-6) + (-7) - 11

12 - 1 = 11

11 - 11 = 0

Which expression is equivalent to this polynomial expression?

Answers

Answer:

Where is the expression

Answer:

Its c :)

Step-by-step explanation:

plato

Solve using a proportion.

Answers

Answer:

i cant see it

Step-by-step explanation:

2ft

For every foot it needs to be 12 ft. So 1 3/4 is 1 foot and 9 inches. Cause we have the whole number which is 1 and 3/4. 3/4 isn’t a whole number it’s a fraction of 12. 1/4 if 12 is 3 cause 12 split into 4 parts is 3. Now we have 3/4 which is 3 of the 1/4ths. We said 1/4 is 3 and if we have three then we multiply 3 x 3 and get 9.

So

1 ft 9 in

But it says to round to the nearest integer so 1 turns to 2.

Hope this helps

Find the measure of D
E
3x
2x

A
B
С

Answers

Answer:

2x+3x+x= 180

6x= 180

x=180/6=30

<DBE = 3x = 3*30 = 90

Step-by-step explanation:

Pls mark as brainliest

Calculus, derivatives. Please help! Show work, if possible. Thanks! :)

Answers

Answer:

y=-x+pi/3+sqrt(3)/2

Step-by-step explanation:

*I assume you are asking for help on part a because the other question is a calculator question.*

We want to find tangent line to y=2sin(x)cos(x) at x=pi/3.

First I prefer to use y=sin(2x). (Double angle identity: sin(2x)=2sin(x)cos(x) .)

Differentiate using chain rule:

y'=2cos(2x)

y' gives us the slope for any tangent along our curve.

So at x=pi/3, the slope is 2cos(2×pi/3)=2cos(2pi/3)=2(-1/2)=-1.

We also need to know a point our line to find the equation of our line.

At x=pi/3, y=sin(2×pi/3)=sin(2pi/3)=sqrt(3)/2.

So a point on our line is (pi/3, sqrt(3)/2).

So by point-slope form of a line we have

y-y1=m(x-x1)

y-sqrt(3)/2=-1(x-pi/3)

Distribute:

y-sqrt(3)/2=-x+pi/3

Add sqrt(3)/2 on both sides:

y=-x+pi/3+sqrt(3)/2

Answer:

y= -x+(pi/3)+(sqrt3)/2

Step-by-step explanation:

The given is y=2sin(x)*cos(x)

to take the derivative we will need to use the constant rule (which says when taking the derivative of any constant multiplied by an expression you take the derivative of the function and multiply it by the constant) and the multiplication rule to get

y'=2(sin(x)*-sin(x)+cos(x)*cos(x))

this would simplify to be

y'=-2sin^2(x)+2cos^2(x)

We then substitute in the given x value to find the slope of the tangent line at that point and get

-2sin^2(pi/3)+2cos^2(pi/3)

which simplifies to

-2*3/4+2*1/4

which is -1

Since we have the slope we need to find the y value to go with the original x so that we have a point to use in point slope form. When we substitute pi/3 into the given equation we get (sqrt 3)/2 and have the point (pi/3, sqrt3/2)

now we write the point slope form equation

y-(sqrt3)/2= -(x-pi/3)

y-(sqrt 3)/2 = -x+pi/3

y= -x+pi/3+(sqrt3)/2

you could simplify the last two terms, but I think it is simplest to understand this way.

For b, enter both equations into graphing software or a calculator

3409/14 with decimal

Answers

Answer:

243.5

Step-by-step explanation:

3409/14 times 7/7

By the way i divided both numerator and denominator by 77 because of gcf

Reduce

487/2

243.5

Define the function
3
g(x) = x^3+ x
. If
f(x) = g^-1(x)
and
f(2) = 1
what is the value of
f'(2)
?

Answers

Given that

[tex]g(x) = x^3 + x[/tex]

the inverse [tex]g^{-1}(x)[/tex] is such that

[tex]g\left(g^{-1}(x)\right) = g^{-1}(x)^3 + g^{-1}(x) = x[/tex]

or

[tex]g\left(f(x)\right) = f(x)^3 + f(x) = x[/tex]

Differentiating both sides using the chain rule gives

[tex]3f(x)^2f'(x) + f'(x) = 1 \\\\ f'(x) \left(3f(x)^2+1\right) = 1 \\\\ f'(x) = \dfrac1{3f(x)^2+1}[/tex]

Then the derivative of f at 2 is

[tex]f'(2) = \dfrac1{3f(2)^2+1} = \boxed{\dfrac14}[/tex]

Tìm ma trận X thỏa:1264X315

Answers

pogpogii mooooooooooo

GE bisects angle DGF , m angle DGE = 53º. Find m angle DGF.

M angle DGF

Answers

Answer:

4.5 degres

Step-by-step explanation:

simplify the expression 6y-2y(x+1)​

Answers

Answer:

2. 4y-2xy

Step-by-step explanation:

6y-2y(x+1)=6y-2xy-2y

=4y-2xy

Micah is saving to buy a new guitar and every month is putting $45 from his paycheck into his savings
account. He has already saved $120. How many more months will it take before he has enough to buy
the guitar if it costs $415?

Answers

Answer:

6.5555 months

Step-by-step explanation:

$415-$120= $295

Micah will need $295 from his paycheck earnings to be able to afford the guitar.

$295/$45= 6.5555months

4[3(5+2)]=95-4x3 is this statement true or false

Answers

Answer:

TRUEeeee

Step-by-step explanation:

bec. there beetwen have a

why is this an impossible sequence to solve?
3x + 1 ?

thanks​

Answers

Answer:

[tex]T(x)=\begin{cases}\frac{3x+1}{2} & x\text{ is odd} \\ \frac{x}{2} & x\text{ is even}\end{cases} \\ for \: instance \: start \: with \: 7 \: \\ which \: is \: an \: odd \: number \\ \frac{21 + 1}{2} = \frac{22}{2} = 11 \\ 11 \: is \: odd \\ \frac{11 + 1}{2} = 6\\ 6 \: is \: even \\ \frac{6}{2} = 3 \\ 3\: is \: odd \\ \frac{9 + 1}{2} = 5 \: \\ 5 \: is \: odd\\ \frac{15 + 1}{2} = 8 \\ 8 \: is \: even \\ \frac{8}{2} = 4 \\ 4 \: is \: even \\ \frac{4}{2} = 2 \: then \: 1 \\ [/tex]

Step-by-step explanation:

Can we connect to 1 for all numbers?

A general proof for a simple pattern seems almost impossible, probably a new branch of mathematics maybe needed to tackle this conjecture...

2. At the gym, Alex spends 24 minutes doing
resistance training for every 30 minutes
spent doing cardio exercises, Carisa spends
15 minutes on resistance for every
20 minutes on cardio, and Manuel spends
14 minutes on resistance for every
15 minutes on cardio. Which person has the
greatest ratio of minutes spent on resistance
to minutes spent on cardio? (Example 2)
Alex
Resistance (min)
Cardio (min)
Carisa
Resistance (min)
Cardio (min)
Manuel
Resistance (min)
Cardio (min)

Answers

Answer:

Manuel

Step-by-step explanation:

You should set up a proportion graph first such as r:c where r is the amount of minutes spent on resistance and c is the amount of minutes spent on cardio.

Alex's ratio: 24:30

Carisa's ratio: 15:20

Manuel's ratio: 14:15

Now you have to find the lowest common multiple (lcm) of the cardio minutes. Their lcm is 60. Now the ratios look like this:

Alex's ratio: 48:60

Carisa's ratio: 45:60

Manuel's ratio: 56:60

The ratio with the highest first number is considered larger. In this case its Manuel since he has the number 56.


The value of coin A is twice the value of coin B. Together, the total value of the two coins is $30.00 What is the value of each coin?

Answers

Answer: Coin A is $20, coin B is $10

Explanation:

A + B = $30

B = $10

$10 x 2 = $20

A = $20

$20 + $10 = $30

Li believes that the graph shows a direct variation.
do +
LI
Why is Li incorrect in saying that the graph shows a direct variation?

Answers

Answer:

b. or 2. When the x-value is 0, the y-value is 1.

Step-by-step explanation:

welcome bestie <3.

NEED HELP FOR REAL 69 points!!!!!!!!!!!!!! ASAP
The physicist from the last problem lets the particle fall another 100 times, and finds that the average travel time is now 16 sec. Remember that when running the simulation many times it took 15 seconds, on average, for the particle to fall to the bottom, with a standard deviation of 1.5 seconds. What does she conclude and why?

Answers

It concludes that newtons laws of motion are at work

Solve for x:
(x + 4) (x - 3) = 0
A. x = -4 and x = 3
B. x= -1 and x = 12
C. x= 1 and x = -12
D. X= 4 and x = -3

Answers

Answer:

answer is A x = -4 and x = 3

Step-by-step explanation:

-4 , 3

switch the signs

Answer:

The answer is "A."

Step-by-step explanation:

x+4=0. x-3=0

x=0-4. x=0+3

x= -4. x= 3

x = -4 and x = 3

The Earth rotates at a unit rate of
.25 degrees per minute. How much
does the earth rotate in half of an
hour?

Answers

Answer:

it will rotate 7.5 degrees

Step-by-step explanation:

T(d) is a function that relates the number of tickets sold for a movie to the number of days since the movie was released. The average rate of change in T(d) for the interval d = 4 and d = 10 is 0. Which statement must be true?The same number of tickets was sold on the fourth day and tenth day.No tickets were sold on the fourth day and tenth day.Fewer tickets were sold on the fourth day than on the tenth day.More tickets were sold on the fourth day than on the tenth day.

Answers

Answer:

The same number of tickets was sold on the fourth day and tenth day.

Step-by-step explanation:

Given, T(d) is a function that relates the number of tickets sold for a movie to the number of days since the movie was  released.

The average rate of change in a function with respect to the independent variable gives the value of change in a particular period.

If average rate of change in T(d) for the interval d = 4 and d = 10 is  is 0 , that means nothing has been changed in the number of tickets sold on 4th day and 10th day after the film release.

Hence, the correct statement is "The same number of tickets was sold on the fourth day and tenth day."

Answer:

In simpler terms, for the edge answer;

A. The same number of tickets was sold on the fourth day and tenth day.

- _ -   long answers scare me...

Step-by-step explanation:

edg- answers ;p

Other Questions
If I could get help Help please. I give brainliest. (I accidentally marked this as mathematics its actually history) i need help on math please solve this If I gave you the word successful does that word have a prefix or suffix? What would it be? Each number in a list of numbers is 3 more than the number before it. The first number in the list is 4. What is the fifth number? Someone plz help me :( Evaluate 8 + 10 4 (2).options:283204 how can you tell that a website is valid and reliable?A. Make sure it sounds valid and reliable when you read it?B. Use the CARS checklistC. If it is written using scholarly terms, then it must be written by an expertD. if there are no grammar or spelling errors then it probably is good 4 digit number that is not divisoble by 2,3,5 or 10 Please help!! Find the length of the third side. If necessary, write in simplest radical form. What should you do when you feel like you are all alone?A. Give in to that negative thought and feeling.B. Realize you are on the Stage of Life and scene ismeant to "look" like you are alone.C. Get upset. using the standard enthalpies of formation, what is the standard enthalpy of reaction? How were the Maya and Aztec civilizations similar? A. They both lacked a strong central government.B. They both developed complex mathematical and calendar systems. C. They both used weapons superior to those of early eastern civilizations D. These civilizations produced few cultural achievements OMG please help me! Please! For school, in art, we have to collect photos of us from years ago. Im contemplating whether or not I should use this one. What do you guys think and why? Thanks! Which of the following statements about the election of 1796 are true If an athletic field has 440 metre, how many rounds must an athlete do to cover a half marathon, I.e 21km and 200 metre guys pls help me!!!!!!! Evaluate the following integral: Que pas en el accidente?